1answer.
Ask question
Login Signup
Ask question
All categories
  • English
  • Mathematics
  • Social Studies
  • Business
  • History
  • Health
  • Geography
  • Biology
  • Physics
  • Chemistry
  • Computers and Technology
  • Arts
  • World Languages
  • Spanish
  • French
  • German
  • Advanced Placement (AP)
  • SAT
  • Medicine
  • Law
  • Engineering
Tju [1.3M]
3 years ago
9

find the height difference between the top of mt Everest 8848 metres and the Mariana Trench 10911 metresbelow sea level

Mathematics
1 answer:
zhuklara [117]3 years ago
6 0
Mount\ Everest\to+8,848m\\\\Mariana\ Trench\to-10,911m\\\\Difference:|-10,911-8,848m|=|-19,759|\\\\\center={\boxed{19,759\ (m)}}\leftarrow answer
You might be interested in
Which of the following equations is proportional? A. y=5x+6 B. y=8x+1 C. y=8x+0 D.y=5x+8
Dmitriy789 [7]

Answer:

pretty sure it is c but might wrong

Step-by-step explanation:

4 0
2 years ago
Expand the logarithmic expression. I will award brainliest.
Natasha_Volkova [10]
\bf \textit{logarithm of factors}\\\\
log_a(xy)\implies log_a(x)+log_a(y)
\\\\\\
\textit{Logarithm of rationals}\\\\
log_a\left(  \frac{x}{y}\right)\implies log_a(x)-log_a(y)
\\\\\\
\textit{Logarithm of exponentials}\\\\
log_a\left( x^  b \right)\implies   b\cdot  log_a(x)\\\\
-------------------------------\\\\
log_3\left( \cfrac{2r^5}{y^8} \right)\implies log_3(2r^5)-log(y^8)
\\\\\\
\boxed{log_3(2)+log_3(r^5)}-log_3(y^8)\implies log_3(2)+5log_3(r)-8log_3(y)
8 0
2 years ago
32 divided by 1.6 how to work out
neonofarm [45]

Answer:

20

Step-by-step explanation:

\frac{32}{1.6} =\frac{32}{\frac{16}{10}} =32*\frac{10}{16} =\frac{320}{16} =20

So the answer is 20

4 0
1 year ago
The profit earned by a hot dog stand is a linear function of the number of hot dogs sold. It costs the owner $48 dollars each mo
aev [14]

Answer: C. y=2x-48

Step-by-step explanation:

8 0
3 years ago
Read 2 more answers
Can someone pls help me with this?
Sunny_sXe [5.5K]

Answer:

the answer is d

Step-by-step explanation:

so for one do y-intercept has to be at (0,25) so we can eliminate answer choices B and C

the reason why it is not a is because a is moving out of constant rate of 25 when it needs to be moving at a constant rate like 10 D is the answer because it's moving at a constant rate of 10 and it has a y intercept of (0,25)

3 0
2 years ago
Read 2 more answers
Other questions:
  • Solve for a <br><br> . 1/2a = 11
    13·2 answers
  • X^2y^8/x^7y^6 if x= -2 and y= 4
    5·1 answer
  • The original price of an item is $35. Complete the table to calculate 21% OF the original price
    14·1 answer
  • EASY 35 POINTS!
    5·1 answer
  • Help!!! Solve for x!! Pls!!!
    11·2 answers
  • !!!PLEASE ANSWER!!!
    8·1 answer
  • a cylindrical tank is a diameter of 14 m and a height of 3.5 m find the capacity of the water tank in litres​
    7·1 answer
  • Help thank you
    6·1 answer
  • What number makes this ratio equivalent?<br><br><br>12/20 = 6/x<br>I will give crown for answer
    5·2 answers
  • The width of a rectangular field is 20 feet less than its length. the area of the field is 12,000 ft2. what is the length of the
    9·1 answer
Add answer
Login
Not registered? Fast signup
Signup
Login Signup
Ask question!